An object has the shape of a square and has side length . Light beams are shone on the object from a big machine. If is the mass of the object, is the power per unit area of the photons, is the speed of light, and is the acceleration of gravity, prove that the minimum value of such that the bar levitates due to the light beams is
Note: This problem was originally proposed by Trung Phan for the IPhOO.
Easy Math Editor
This discussion board is a place to discuss our Daily Challenges and the math and science related to those challenges. Explanations are more than just a solution — they should explain the steps and thinking strategies that you used to obtain the solution. Comments should further the discussion of math and science.
When posting on Brilliant:
*italics*
or_italics_
**bold**
or__bold__
paragraph 1
paragraph 2
[example link](https://brilliant.org)
> This is a quote
\(
...\)
or\[
...\]
to ensure proper formatting.2 \times 3
2^{34}
a_{i-1}
\frac{2}{3}
\sqrt{2}
\sum_{i=1}^3
\sin \theta
\boxed{123}
Comments
Are the photons striking perpendicularly? Are they rebounding? (Please clarify.)
If both of above assumptions are valid, using classical mechanics, I get P=2a2cmg
Log in to reply
This is the issue. We did not mention any assumptions!
each photon (assuming it strikes and rebounds perpendicularly) grants a momentum of 2E/c E=2Pta^2/c so we need mgt=2E/c=2Pta^2/c so P=cmg/2a^2
Ahaan, can you suggest something Jatin and I might be missing?
Log in to reply
We gave a statement to prove and said nothing about how the light is reflected. Some investigation should suggest that it's reflected in the way that the probability for the ray to scatter in any direction is equal.
Log in to reply
Hey, ahaan, even if we assume that the incidence and scattering is random, we get P=4a2πcmg, not 5a24cmg
Initial vertical momentum/sec = cPa2∫−2π2πcosθπdθ=πc2Pa2.
Final vertical momentum/sec = −πc2Pa2
Hence, Force = πc4Pa2
Hence, Pmin=4a2πcmg, which is quite close to the answer you show, but not exactly equal!
Log in to reply
@jatin yadav and all others, sorry to bother you here. How much are you expecting in mains Jatin?
Right, so, given that it's unspecified we are free to think of whatever arrangement for the geometry of the light shining onto the surface, right?
Light shining perpendicularly onto the square results in the most efficient momentum transfer to the square.
Like Jatin said, under that assumption (that the light shines perpendicularly), it is possible the levitate the bar with P=2a2cmg which is less than 5a24cmg, contradicting the claim in the problem that P=2a2cmg is the minimum power that can achieve levitation.
The only possibilities are that Jatin and I are reading the problem wrong, or there is something wrong with our argument, or possibly that the claim is wrong. Can you perhaps point out where we might be going wrong in our reading of the problem?
Log in to reply
Which is why we scored everybody who got a 21-coefficient 5/7. I will come up with a better reasoning soon.